IASbaba Prelims 60 Days Plan, Rapid Revision Series (RaRe)
Archives
Hello Friends
The 60 Days Rapid Revision (RaRe) Series is IASbaba’s Flagship Initiative recommended by Toppers and loved by the aspirants’ community every year.
It is the most comprehensive program which will help you complete the syllabus, revise and practice tests on a daily basis. The Programme on a daily basis includes
Daily Prelims MCQs from Static (Monday – Saturday)
- Daily Static Quiz will cover all the topics of static subjects – Polity, History, Geography, Economics, Environment and Science and technology.
- 20 questions will be posted daily and these questions are framed from the topics mentioned in the schedule.
- It will ensure timely and streamlined revision of your static subjects.
Daily Current Affairs MCQs (Monday – Saturday)
- Daily 5 Current Affairs questions, based on sources like ‘The Hindu’, ‘Indian Express’ and ‘PIB’, would be published from Monday to Saturday according to the schedule.
Daily CSAT Quiz (Monday – Friday)
- CSAT has been an Achilles heel for many aspirants.
- Daily 5 CSAT Questions will be published.
Note – Daily Test of 20 static questions, 10 current affairs, and 5 CSAT questions. (35 Prelims Questions) in QUIZ FORMAT will be updated on a daily basis.
To Know More about 60 Days Rapid Revision (RaRe) Series – CLICK HERE
60 Day Rapid Revision (RaRe) Series Schedule – CLICK HERE
Important Note
- Comment your Scores in the Comment Section. This will keep you accountable, responsible and sincere in days to come.
- It will help us come out with the Cut-Off on a Daily Basis.
- Let us know if you enjoyed today’s test 🙂
- You can post your comments in the given format
- (1) Your Score
- (2) Matrix Meter
- (3) New Learning from the Test
Test-summary
0 of 35 questions completed
Questions:
- 1
- 2
- 3
- 4
- 5
- 6
- 7
- 8
- 9
- 10
- 11
- 12
- 13
- 14
- 15
- 16
- 17
- 18
- 19
- 20
- 21
- 22
- 23
- 24
- 25
- 26
- 27
- 28
- 29
- 30
- 31
- 32
- 33
- 34
- 35
Information
The following Test is based on the syllabus of 60 Days Plan-2025 for UPSC IAS Prelims 2025.
To view Solutions, follow these instructions:
- Click on – ‘Start Test’ button
- Solve Questions
- Click on ‘Test Summary’ button
- Click on ‘Finish Test’ button
- Now click on ‘View Questions’ button – here you will see solutions and links.
You have already completed the test before. Hence you can not start it again.
Test is loading...
You must sign in or sign up to start the test.
You have to finish following test, to start this test:
Results
0 of 35 questions answered correctly
Your time:
Time has elapsed
You have scored 0 points out of 0 points, (0)
Average score |
|
Your score |
|
Categories
- Not categorized 0%
Pos. | Name | Entered on | Points | Result |
---|---|---|---|---|
Table is loading | ||||
No data available | ||||
- 1
- 2
- 3
- 4
- 5
- 6
- 7
- 8
- 9
- 10
- 11
- 12
- 13
- 14
- 15
- 16
- 17
- 18
- 19
- 20
- 21
- 22
- 23
- 24
- 25
- 26
- 27
- 28
- 29
- 30
- 31
- 32
- 33
- 34
- 35
- Answered
- Review
-
Question 1 of 35
1. Question
Consider the following statements with reference to ‘Red Sanders’:
- It is listed in the ‘endangered’ category in its Red List of IUCN.
- It is aromatic in nature.
- The species is endemic to North-eastern part of India.
Which of the statements above is/are correct?
Correct
Solution (a)
Explanation:
- The Red Sanders (Pterocarpus santalinus), also known as Red Sandalwood, is classified as Endangered on the International Union for Conservation of Nature (IUCN) Red List. The species faces a high risk of extinction in the wild due to overexploitation for its valuable red-colored wood, illegal logging, and habitat degradation. (Statement 1 is correct)
- Red Sanders is not aromatic. It is often confused with Sandalwood (Santalum album), which is highly valued for its fragrant heartwood and essential oils. Red Sanders is prized for its deep red timber, used in fine woodworking, musical instruments, and natural dyes, but it does not possess aromatic properties. (Statement 2 is incorrect)
- Red Sanders is endemic to the southern parts of India, specifically the Eastern Ghats region in the state of Andhra Pradesh, with some occurrences in Tamil Nadu and Karnataka. It does not naturally occur in the north-eastern part of India. (Statement 3 is incorrect)
Incorrect
Solution (a)
Explanation:
- The Red Sanders (Pterocarpus santalinus), also known as Red Sandalwood, is classified as Endangered on the International Union for Conservation of Nature (IUCN) Red List. The species faces a high risk of extinction in the wild due to overexploitation for its valuable red-colored wood, illegal logging, and habitat degradation. (Statement 1 is correct)
- Red Sanders is not aromatic. It is often confused with Sandalwood (Santalum album), which is highly valued for its fragrant heartwood and essential oils. Red Sanders is prized for its deep red timber, used in fine woodworking, musical instruments, and natural dyes, but it does not possess aromatic properties. (Statement 2 is incorrect)
- Red Sanders is endemic to the southern parts of India, specifically the Eastern Ghats region in the state of Andhra Pradesh, with some occurrences in Tamil Nadu and Karnataka. It does not naturally occur in the north-eastern part of India. (Statement 3 is incorrect)
-
Question 2 of 35
2. Question
With reference to the IUCN Red List, which of the following statements is/are correct?
- The species groups covered in the list so far are biased towards forest ecosystems.
- All taxonomic groups have been completely assessed under the list.
Select the answer using the code given below:
Correct
Solution (a)
Explanation:
- The IUCN Red List acknowledges that there is a bias towards terrestrial species, particularly those in forest ecosystems. This means that species inhabiting forests are more extensively assessed compared to those in other ecosystems like freshwater or marine environments. This bias exists due to various factors, including the accessibility of habitats, funding priorities, and the level of scientific research conducted in different ecosystems. (Statement 1 is correct)
- The IUCN Red List has not completely assessed all taxonomic groups. While vertebrates like mammals, birds, and amphibians have been comprehensively evaluated, many other groups—such as invertebrates, plants, fungi, and microorganisms—remain underrepresented. The vast diversity of life makes it challenging to assess every species, and there are ongoing efforts to expand assessments across all taxonomic groups. (Statement 2 is incorrect)
Incorrect
Solution (a)
Explanation:
- The IUCN Red List acknowledges that there is a bias towards terrestrial species, particularly those in forest ecosystems. This means that species inhabiting forests are more extensively assessed compared to those in other ecosystems like freshwater or marine environments. This bias exists due to various factors, including the accessibility of habitats, funding priorities, and the level of scientific research conducted in different ecosystems. (Statement 1 is correct)
- The IUCN Red List has not completely assessed all taxonomic groups. While vertebrates like mammals, birds, and amphibians have been comprehensively evaluated, many other groups—such as invertebrates, plants, fungi, and microorganisms—remain underrepresented. The vast diversity of life makes it challenging to assess every species, and there are ongoing efforts to expand assessments across all taxonomic groups. (Statement 2 is incorrect)
-
Question 3 of 35
3. Question
Consider the following animals:
- Bombardier Beetle
- Spitting Cobra
- Malabar Pit Viper
Which of the above organisms can eject substances as a defence mechanism against predators?
Correct
Solution (a)
Explanation:
- The Bombardier Beetle is known for its unique defense mechanism where it ejects a hot, noxious chemical spray from the tip of its abdomen to deter predators. The beetle mixes chemical compounds (hydroquinones and hydrogen peroxide) stored in its body, causing a violent exothermic reaction that releases the boiling, toxic spray. (Statement 1 is correct)
- The Spitting Cobra is a group of cobras capable of ejecting venom from their fangs when threatened. In India, species like the Monocled Cobra (Naja kaouthia) are known to exhibit spitting behavior. The venom is directed towards the eyes of the predator, causing pain and potential temporary or permanent blindness. (Statement 2 is correct)
The Malabar Pit Viper is a venomous snake endemic to the Western Ghats of India. It relies on camouflage and ambush predation but does not eject substances as a defense mechanism. Instead, it bites to inject venom when threatened or when capturing prey. The Malabar Pit Viper is not known to spit or spray venom. (Statement 3 is incorrect)
Incorrect
Solution (a)
Explanation:
- The Bombardier Beetle is known for its unique defense mechanism where it ejects a hot, noxious chemical spray from the tip of its abdomen to deter predators. The beetle mixes chemical compounds (hydroquinones and hydrogen peroxide) stored in its body, causing a violent exothermic reaction that releases the boiling, toxic spray. (Statement 1 is correct)
- The Spitting Cobra is a group of cobras capable of ejecting venom from their fangs when threatened. In India, species like the Monocled Cobra (Naja kaouthia) are known to exhibit spitting behavior. The venom is directed towards the eyes of the predator, causing pain and potential temporary or permanent blindness. (Statement 2 is correct)
The Malabar Pit Viper is a venomous snake endemic to the Western Ghats of India. It relies on camouflage and ambush predation but does not eject substances as a defense mechanism. Instead, it bites to inject venom when threatened or when capturing prey. The Malabar Pit Viper is not known to spit or spray venom. (Statement 3 is incorrect)
-
Question 4 of 35
4. Question
This species constructs a nest throughout the year. It is found in and around Manas National Park and needs relatively undisturbed, tall ‘terai’ grasslands. It is in the critically endangered group of IUCN Red list. There is a parasite which only feeds on it and thus, it also comes under the critically endangered grouping.
Based on the description given above, identify the species from the following options:
Correct
Solution (d)
Explanation:
- The Bengal Florican (Houbaropsis bengalensis) is a large bird species also found in the grasslands of Manas National Park and listed as Critically Endangered. While it inhabits tall grasslands, it does not construct nests throughout the year. Instead, it nests seasonally on the ground during the breeding season. There is no known parasite exclusively dependent on the Bengal Florican.
- The Greater Adjutant Stork (Leptoptilos dubius) is a large wading bird found in parts of Assam, including areas near Manas National Park. It is classified as Endangered, not Critically Endangered. This species nests in tall trees, not in grasslands, and does not construct nests throughout the year.
- The Swamp Deer (Rucervus duvaucelii), also known as Barasingha, inhabits swampy grasslands in northern India, including parts of Assam. It is listed as Vulnerable on the IUCN Red List, not Critically Endangered. Swamp Deer do not construct nests; they are grazing mammals. There is no known parasite that feeds exclusively on them.
- The Pygmy Hog (Porcula salvania) is the smallest and rarest wild pig species in the world, listed as Critically Endangered on the IUCN Red List. It is endemic to the Terai grasslands at the foothills of the Himalayas, primarily in and around Manas National Park in Assam, India. Unique among pigs, the Pygmy Hog constructs nests throughout the year using grasses to create a dome-shaped structure for shelter.
- The Pygmy Hog is host to a parasitic louse species called Haematopinus oliveri, which feeds exclusively on it. Since the parasite depends entirely on the Pygmy Hog for survival, it is also classified as Critically Endangered. (Option (d) is correct)
Incorrect
Solution (d)
Explanation:
- The Bengal Florican (Houbaropsis bengalensis) is a large bird species also found in the grasslands of Manas National Park and listed as Critically Endangered. While it inhabits tall grasslands, it does not construct nests throughout the year. Instead, it nests seasonally on the ground during the breeding season. There is no known parasite exclusively dependent on the Bengal Florican.
- The Greater Adjutant Stork (Leptoptilos dubius) is a large wading bird found in parts of Assam, including areas near Manas National Park. It is classified as Endangered, not Critically Endangered. This species nests in tall trees, not in grasslands, and does not construct nests throughout the year.
- The Swamp Deer (Rucervus duvaucelii), also known as Barasingha, inhabits swampy grasslands in northern India, including parts of Assam. It is listed as Vulnerable on the IUCN Red List, not Critically Endangered. Swamp Deer do not construct nests; they are grazing mammals. There is no known parasite that feeds exclusively on them.
- The Pygmy Hog (Porcula salvania) is the smallest and rarest wild pig species in the world, listed as Critically Endangered on the IUCN Red List. It is endemic to the Terai grasslands at the foothills of the Himalayas, primarily in and around Manas National Park in Assam, India. Unique among pigs, the Pygmy Hog constructs nests throughout the year using grasses to create a dome-shaped structure for shelter.
- The Pygmy Hog is host to a parasitic louse species called Haematopinus oliveri, which feeds exclusively on it. Since the parasite depends entirely on the Pygmy Hog for survival, it is also classified as Critically Endangered. (Option (d) is correct)
-
Question 5 of 35
5. Question
With reference to differences between African Cheetah and Asiatic Cheetah, consider the following statements:
- Asiatic cheetahs are slightly larger in size than African cheetahs.
- Asiatic cheetahs have a thicker coat and more powerful neck than African cheetahs.
- Both are listed as critically endangered species in the IUCN Red list.
How many of the statements above are correct?
Correct
Solution (a)
Explanation:
- The Asiatic cheetah (Acinonyx jubatus venaticus) is generally smaller than the African cheetah. Asiatic cheetahs have a more slender build and are adapted to the arid and semi-arid environments of Iran. Their smaller size is an adaptation to the scarcity of prey and harsher conditions compared to the habitats of African cheetahs. (Statement 1 is incorrect)
- Asiatic cheetahs have developed a thicker coat to adapt to the colder climates of their current range in Iran. They also possess a more powerful neck, which may assist them in hunting larger prey in their environment. These physical differences are adaptations to their specific habitat conditions. (Statement 2 is correct)
- The Asiatic cheetah is listed as Critically Endangered on the IUCN Red List due to its extremely low population, estimated to be fewer than 50 individuals confined to Iran. The African cheetah, however, is listed as Vulnerable, not Critically Endangered. While the African cheetah population is decreasing, it remains more widely distributed across several African countries. (Statement 3 is incorrect)
Incorrect
Solution (a)
Explanation:
- The Asiatic cheetah (Acinonyx jubatus venaticus) is generally smaller than the African cheetah. Asiatic cheetahs have a more slender build and are adapted to the arid and semi-arid environments of Iran. Their smaller size is an adaptation to the scarcity of prey and harsher conditions compared to the habitats of African cheetahs. (Statement 1 is incorrect)
- Asiatic cheetahs have developed a thicker coat to adapt to the colder climates of their current range in Iran. They also possess a more powerful neck, which may assist them in hunting larger prey in their environment. These physical differences are adaptations to their specific habitat conditions. (Statement 2 is correct)
- The Asiatic cheetah is listed as Critically Endangered on the IUCN Red List due to its extremely low population, estimated to be fewer than 50 individuals confined to Iran. The African cheetah, however, is listed as Vulnerable, not Critically Endangered. While the African cheetah population is decreasing, it remains more widely distributed across several African countries. (Statement 3 is incorrect)
-
Question 6 of 35
6. Question
Consider the following conservation areas/regions:
- Singalila National Park,
- Neora Valley National Park
- Jalda Para National Park
- Gorumara National Park
Which of the above areas have the presence of Red Pandas?
Correct
Solution (a)
Explanation:
- Singalila National Park, located in the Darjeeling district of West Bengal, is one of the prime habitats of the Red Panda (Ailurus fulgens) in India. The park’s high-altitude temperate forests with abundant bamboo make it an ideal environment for Red Pandas. Conservation efforts are ongoing in this park to protect and study this elusive species.
- Neora Valley National Park, situated in the Kalimpong district of West Bengal, is another significant habitat for Red Pandas in India. The park’s pristine forests and rich biodiversity provide a safe haven for Red Pandas. It is one of the least disturbed habitats and supports a healthy population of this endangered species. (Option (a) is correct)
- Jaldapara National Park is located in the Alipurduar District of West Bengal. The park is known for its grasslands and forests, supporting species like the Indian one-horned rhinoceros, elephants, and various deer species. However, it does not provide the high-altitude, temperate forest habitat required by Red Pandas.
- Gorumara National Park is situated in the Jalpaiguri District of West Bengal, in the foothills of the Eastern Himalayas. The park is primarily composed of grasslands and tropical forests, housing species like the Indian rhinoceros, elephants, and bison. It lacks the temperate forest ecosystem that Red Pandas require.
Incorrect
Solution (a)
Explanation:
- Singalila National Park, located in the Darjeeling district of West Bengal, is one of the prime habitats of the Red Panda (Ailurus fulgens) in India. The park’s high-altitude temperate forests with abundant bamboo make it an ideal environment for Red Pandas. Conservation efforts are ongoing in this park to protect and study this elusive species.
- Neora Valley National Park, situated in the Kalimpong district of West Bengal, is another significant habitat for Red Pandas in India. The park’s pristine forests and rich biodiversity provide a safe haven for Red Pandas. It is one of the least disturbed habitats and supports a healthy population of this endangered species. (Option (a) is correct)
- Jaldapara National Park is located in the Alipurduar District of West Bengal. The park is known for its grasslands and forests, supporting species like the Indian one-horned rhinoceros, elephants, and various deer species. However, it does not provide the high-altitude, temperate forest habitat required by Red Pandas.
- Gorumara National Park is situated in the Jalpaiguri District of West Bengal, in the foothills of the Eastern Himalayas. The park is primarily composed of grasslands and tropical forests, housing species like the Indian rhinoceros, elephants, and bison. It lacks the temperate forest ecosystem that Red Pandas require.
-
Question 7 of 35
7. Question
Consider the following statements with reference to Indian Pangolin:
- They are scaly anteater mammals.
- It is widely distributed in India.
- Its IUCN status is endangered.
How many of the statements above are correct?
Correct
Solution (c)
Explanation:
- The Indian Pangolin (Manis crassicaudata) is a mammal known for its unique covering of protective keratin scales, which makes it the only mammal entirely covered in scales. These scales act as armor against predators. The Indian Pangolin is often referred to as a “scaly anteater” because it feeds primarily on ants and termites, using its long, sticky tongue to capture them. It has no teeth and relies on a muscular stomach to grind its food.
- The Indian Pangolin is widely distributed across the Indian subcontinent. In India, it is found throughout the country south of the Himalayas, except the northeastern region. Its habitat includes a range of environments such as forests (both tropical and subtropical), grasslands, and agricultural lands. Despite its wide distribution, the species is difficult to spot due to its solitary and nocturnal nature.
The Indian Pangolin is listed as Endangered on the IUCN Red List of Threatened Species. The population of the Indian Pangolin has been declining rapidly due to illegal hunting and poaching for its meat and scales. The scales are highly valued in traditional medicine, particularly in East Asia. Additionally, habitat loss due to deforestation and land-use changes has contributed to its endangered status. (Option (c) is correct)
Incorrect
Solution (c)
Explanation:
- The Indian Pangolin (Manis crassicaudata) is a mammal known for its unique covering of protective keratin scales, which makes it the only mammal entirely covered in scales. These scales act as armor against predators. The Indian Pangolin is often referred to as a “scaly anteater” because it feeds primarily on ants and termites, using its long, sticky tongue to capture them. It has no teeth and relies on a muscular stomach to grind its food.
- The Indian Pangolin is widely distributed across the Indian subcontinent. In India, it is found throughout the country south of the Himalayas, except the northeastern region. Its habitat includes a range of environments such as forests (both tropical and subtropical), grasslands, and agricultural lands. Despite its wide distribution, the species is difficult to spot due to its solitary and nocturnal nature.
The Indian Pangolin is listed as Endangered on the IUCN Red List of Threatened Species. The population of the Indian Pangolin has been declining rapidly due to illegal hunting and poaching for its meat and scales. The scales are highly valued in traditional medicine, particularly in East Asia. Additionally, habitat loss due to deforestation and land-use changes has contributed to its endangered status. (Option (c) is correct)
-
Question 8 of 35
8. Question
In which of the following States is the Nilgiri Tahr found in its natural habitat?
- Tamil Nadu
- Kerala
- Karnataka
- Andhra Pradesh
Select the correct answer using the codes given below.
Correct
Solution (a)
Explanation:
- The Nilgiri Tahr (Nilgiritragus hylocrius) is endemic to the Western Ghats of South India. In Tamil Nadu, it is primarily found in the high-altitude grasslands and rocky terrains of the Nilgiri Hills, Anaimalai Hills, and the Palani Hills. The species is adapted to the montane grasslands and is often spotted in protected areas like the Mukkurthi National Park. (Statement 1 is correct)
- In Kerala, the Nilgiri Tahr inhabits the southern portion of the Western Ghats. The most significant population resides in the Eravikulam National Park, near Munnar, which was established primarily for the conservation of this species. Other areas include the Silent Valley National Park and Pampadum Shola National Park. (Statement 2 is correct)
- Although the Western Ghats extend into Karnataka, the Nilgiri Tahr is not found in this state. The species’ natural range does not cover Karnataka due to habitat preferences and geographical barriers. Karnataka’s portion of the Western Ghats does not provide the specific high-altitude grassland habitat that Nilgiri Tahrs require. (Statement 3 is incorrect)
- The Nilgiri Tahr is not present in Andhra Pradesh. The state’s terrain and ecosystems differ from the high-altitude grasslands of the Western Ghats. Andhra Pradesh primarily consists of coastal plains and the Eastern Ghats, which are not suitable habitats for the Nilgiri Tahr. (Statement 4 is incorrect)
Incorrect
Solution (a)
Explanation:
- The Nilgiri Tahr (Nilgiritragus hylocrius) is endemic to the Western Ghats of South India. In Tamil Nadu, it is primarily found in the high-altitude grasslands and rocky terrains of the Nilgiri Hills, Anaimalai Hills, and the Palani Hills. The species is adapted to the montane grasslands and is often spotted in protected areas like the Mukkurthi National Park. (Statement 1 is correct)
- In Kerala, the Nilgiri Tahr inhabits the southern portion of the Western Ghats. The most significant population resides in the Eravikulam National Park, near Munnar, which was established primarily for the conservation of this species. Other areas include the Silent Valley National Park and Pampadum Shola National Park. (Statement 2 is correct)
- Although the Western Ghats extend into Karnataka, the Nilgiri Tahr is not found in this state. The species’ natural range does not cover Karnataka due to habitat preferences and geographical barriers. Karnataka’s portion of the Western Ghats does not provide the specific high-altitude grassland habitat that Nilgiri Tahrs require. (Statement 3 is incorrect)
- The Nilgiri Tahr is not present in Andhra Pradesh. The state’s terrain and ecosystems differ from the high-altitude grasslands of the Western Ghats. Andhra Pradesh primarily consists of coastal plains and the Eastern Ghats, which are not suitable habitats for the Nilgiri Tahr. (Statement 4 is incorrect)
-
Question 9 of 35
9. Question
It is a species of goat-antelope found in the hills of Arunachal Pradesh and Sikkim at an altitude of 2000-3000 metres. It is listed under Schedule I of the Indian Wildlife (Protection) Act, 1972 and is also classified as endangered by the International Union for Conservation of Nature.
Based on the description given above, identify the species from the following:
Correct
Solution (b)
Explanation:
- The Himalayan Serow (Capricornis sumatraensis thar) is a medium-sized goat-antelope found in the Himalayas, including parts of Arunachal Pradesh and Sikkim. It inhabits steep, rugged terrains at elevations ranging from 2000 to 3500 metres. The Himalayan Serow is listed under Schedule I of the Indian Wildlife (Protection) Act, 1972, but it is classified as Vulnerable by the IUCN Red List, not Endangered. While it shares a similar habitat, it does not precisely match the description provided.
- The Mishmi Takin (Budorcas taxicolor taxicolor) is a large goat-antelope native to the Eastern Himalayas in India, particularly in Arunachal Pradesh and parts of Sikkim. It is typically found at altitudes between 2000 and 3000 metres. The Mishmi Takin is listed under Schedule I of the Indian Wildlife (Protection) Act, 1972, providing it the highest level of legal protection. It is also classified as Endangered on the IUCN Red List due to habitat loss and hunting pressures. The species perfectly fits the description given. (Option (b) is correct)
- The Red Goral (Naemorhedus baileyi) is a small goat-antelope found in the forested hills of northeastern India, including Arunachal Pradesh, but at lower elevations ranging from 1000 to 2000 metres. It is listed under Schedule I of the Indian Wildlife (Protection) Act, 1972, but is classified as Vulnerable by the IUCN, not Endangered. Additionally, its smaller size and different altitude range make it inconsistent with the description.
- The Himalayan Tahr (Hemitragus jemlahicus) is a large ungulate found in the Himalayan regions of India, but primarily in the Western Himalayas—in states like Uttarakhand and Himachal Pradesh—not in Arunachal Pradesh or Sikkim. It inhabits elevations from 2500 to 5000 metres, higher than the range specified in the description. The Himalayan Tahr is listed under Schedule III of the Indian Wildlife (Protection) Act, 1972, and is classified as Near Threatened by the IUCN. Therefore, it does not match the criteria provided.
Incorrect
Solution (b)
Explanation:
- The Himalayan Serow (Capricornis sumatraensis thar) is a medium-sized goat-antelope found in the Himalayas, including parts of Arunachal Pradesh and Sikkim. It inhabits steep, rugged terrains at elevations ranging from 2000 to 3500 metres. The Himalayan Serow is listed under Schedule I of the Indian Wildlife (Protection) Act, 1972, but it is classified as Vulnerable by the IUCN Red List, not Endangered. While it shares a similar habitat, it does not precisely match the description provided.
- The Mishmi Takin (Budorcas taxicolor taxicolor) is a large goat-antelope native to the Eastern Himalayas in India, particularly in Arunachal Pradesh and parts of Sikkim. It is typically found at altitudes between 2000 and 3000 metres. The Mishmi Takin is listed under Schedule I of the Indian Wildlife (Protection) Act, 1972, providing it the highest level of legal protection. It is also classified as Endangered on the IUCN Red List due to habitat loss and hunting pressures. The species perfectly fits the description given. (Option (b) is correct)
- The Red Goral (Naemorhedus baileyi) is a small goat-antelope found in the forested hills of northeastern India, including Arunachal Pradesh, but at lower elevations ranging from 1000 to 2000 metres. It is listed under Schedule I of the Indian Wildlife (Protection) Act, 1972, but is classified as Vulnerable by the IUCN, not Endangered. Additionally, its smaller size and different altitude range make it inconsistent with the description.
- The Himalayan Tahr (Hemitragus jemlahicus) is a large ungulate found in the Himalayan regions of India, but primarily in the Western Himalayas—in states like Uttarakhand and Himachal Pradesh—not in Arunachal Pradesh or Sikkim. It inhabits elevations from 2500 to 5000 metres, higher than the range specified in the description. The Himalayan Tahr is listed under Schedule III of the Indian Wildlife (Protection) Act, 1972, and is classified as Near Threatened by the IUCN. Therefore, it does not match the criteria provided.
-
Question 10 of 35
10. Question
With reference to Ganges River Dolphin, consider the following statements:
- This species is also referred to as the ‘Blind Dolphin’.
- Its habitat is spread across the Ganga and Brahmaputra River systems.
- It is listed as critically endangered in the IUCN Red list.
Which of the statements given above is/are correct?
Correct
Solution (b)
Explanation:
- The Ganges River Dolphin (Platanista gangetica gangetica) is commonly known as the ‘Blind Dolphin’ because it has evolved to live in the muddy rivers of the Indian subcontinent and has lost its crystalline eye lens. As a result, its eyes lack a lens, rendering it effectively blind, and it relies on echolocation to navigate and hunt for prey in the turbid river waters. (Statement 1 is correct)
- The Ganges River Dolphin inhabits the Ganga and Brahmaputra River systems, including their tributaries, in India, Nepal, and Bangladesh. The species prefers freshwater rivers with ample depth and slow-moving water, which are characteristic of these river systems. The dolphins are distributed throughout the Ganges-Brahmaputra-Meghna and Karnaphuli-Sangu river systems. (Statement 2 is correct)
- The Ganges River Dolphin is classified as Endangered, not Critically Endangered, on the IUCN Red List of Threatened Species. The species faces several threats, including habitat fragmentation due to dams and barrages, pollution, accidental entanglement in fishing nets (bycatch), and depletion of prey base. While its population is declining, it has not reached the criteria for Critically Endangered status yet. (Statement 3 is incorrect)
Incorrect
Solution (b)
Explanation:
- The Ganges River Dolphin (Platanista gangetica gangetica) is commonly known as the ‘Blind Dolphin’ because it has evolved to live in the muddy rivers of the Indian subcontinent and has lost its crystalline eye lens. As a result, its eyes lack a lens, rendering it effectively blind, and it relies on echolocation to navigate and hunt for prey in the turbid river waters. (Statement 1 is correct)
- The Ganges River Dolphin inhabits the Ganga and Brahmaputra River systems, including their tributaries, in India, Nepal, and Bangladesh. The species prefers freshwater rivers with ample depth and slow-moving water, which are characteristic of these river systems. The dolphins are distributed throughout the Ganges-Brahmaputra-Meghna and Karnaphuli-Sangu river systems. (Statement 2 is correct)
- The Ganges River Dolphin is classified as Endangered, not Critically Endangered, on the IUCN Red List of Threatened Species. The species faces several threats, including habitat fragmentation due to dams and barrages, pollution, accidental entanglement in fishing nets (bycatch), and depletion of prey base. While its population is declining, it has not reached the criteria for Critically Endangered status yet. (Statement 3 is incorrect)
-
Question 11 of 35
11. Question
Consider the following birds:
- Siberian Crane
- Bengal Florican
- Indian Peafowl
How many of the birds above are listed under the critically endangered category of IUCN?
Correct
Solution (b)
Explanation:
- The Siberian Crane (Leucogeranus leucogeranus) is listed as Critically Endangered on the IUCN Red List. This species is known for its long-distance migration, breeding in the wetlands of Siberia and wintering in parts of China and Iran. Historically, a central population used to migrate to India, particularly to Bharatpur in Rajasthan, but this migratory route is now considered extinct due to habitat loss, hunting along migration paths, and environmental changes. (Statement 1 is correct)
- The Bengal Florican (Houbaropsis bengalensis) is also classified as Critically Endangered on the IUCN Red List. It is a bustard species native to the Indian subcontinent, with its population primarily in the grasslands of Uttar Pradesh, Assam, Arunachal Pradesh, and parts of Nepal and Cambodia. The main threats to the Bengal Florican include habitat loss due to agricultural expansion, infrastructure development, and degradation of grasslands. (Statement 2 is correct)
- The Indian Peafowl (Pavo cristatus), the national bird of India, is listed as Least Concern on the IUCN Red List. It is widely distributed across the Indian subcontinent and has a stable population trend. The species adapts well to a variety of habitats, including forests, farmlands, and urban areas. While it may face localized threats from hunting and habitat degradation, it does not meet the criteria for a threatened category at the global level. (Statement 3 is incorrect)
Incorrect
Solution (b)
Explanation:
- The Siberian Crane (Leucogeranus leucogeranus) is listed as Critically Endangered on the IUCN Red List. This species is known for its long-distance migration, breeding in the wetlands of Siberia and wintering in parts of China and Iran. Historically, a central population used to migrate to India, particularly to Bharatpur in Rajasthan, but this migratory route is now considered extinct due to habitat loss, hunting along migration paths, and environmental changes. (Statement 1 is correct)
- The Bengal Florican (Houbaropsis bengalensis) is also classified as Critically Endangered on the IUCN Red List. It is a bustard species native to the Indian subcontinent, with its population primarily in the grasslands of Uttar Pradesh, Assam, Arunachal Pradesh, and parts of Nepal and Cambodia. The main threats to the Bengal Florican include habitat loss due to agricultural expansion, infrastructure development, and degradation of grasslands. (Statement 2 is correct)
- The Indian Peafowl (Pavo cristatus), the national bird of India, is listed as Least Concern on the IUCN Red List. It is widely distributed across the Indian subcontinent and has a stable population trend. The species adapts well to a variety of habitats, including forests, farmlands, and urban areas. While it may face localized threats from hunting and habitat degradation, it does not meet the criteria for a threatened category at the global level. (Statement 3 is incorrect)
-
Question 12 of 35
12. Question
Which of the following animal saw a drastic decrease in its population due to use of a veterinary nonsteroidal anti-inflammatory drug (NSAID)?
Correct
Solution (a)
Explanation:
- The Indian Vulture (Gyps indicus) experienced a catastrophic population decline due to the widespread use of the veterinary NSAID diclofenac. Livestock treated with diclofenac leave behind carcasses containing residues of the drug. When vultures consume these carcasses, they suffer from acute kidney failure, leading to visceral gout and death. This led to a decline of over 95% in vulture populations across the Indian subcontinent during the 1990s and early 2000s. (Option (a) is correct)
- The Asiatic Lion (Panthera leo persica) is found exclusively in the Gir Forest of Gujarat. While it is endangered due to its limited range and threats like poaching, habitat loss, and human-wildlife conflict, its population decline is not linked to the use of veterinary NSAIDs like diclofenac.
- The Indian Rhinoceros (Rhinoceros unicornis), also known as the Greater One-Horned Rhinoceros, faced population declines primarily due to poaching for its horn and habitat loss, not due to NSAID use. Conservation initiatives have led to population recoveries in protected areas like Kaziranga National Park.
- The Bengal Tiger (Panthera tigris tigris) has been threatened by poaching, habitat fragmentation, and human-wildlife conflict, but there is no evidence linking its decline to veterinary NSAIDs. Conservation programs like Project Tiger have been implemented to protect and increase tiger populations in India.
Incorrect
Solution (a)
Explanation:
- The Indian Vulture (Gyps indicus) experienced a catastrophic population decline due to the widespread use of the veterinary NSAID diclofenac. Livestock treated with diclofenac leave behind carcasses containing residues of the drug. When vultures consume these carcasses, they suffer from acute kidney failure, leading to visceral gout and death. This led to a decline of over 95% in vulture populations across the Indian subcontinent during the 1990s and early 2000s. (Option (a) is correct)
- The Asiatic Lion (Panthera leo persica) is found exclusively in the Gir Forest of Gujarat. While it is endangered due to its limited range and threats like poaching, habitat loss, and human-wildlife conflict, its population decline is not linked to the use of veterinary NSAIDs like diclofenac.
- The Indian Rhinoceros (Rhinoceros unicornis), also known as the Greater One-Horned Rhinoceros, faced population declines primarily due to poaching for its horn and habitat loss, not due to NSAID use. Conservation initiatives have led to population recoveries in protected areas like Kaziranga National Park.
- The Bengal Tiger (Panthera tigris tigris) has been threatened by poaching, habitat fragmentation, and human-wildlife conflict, but there is no evidence linking its decline to veterinary NSAIDs. Conservation programs like Project Tiger have been implemented to protect and increase tiger populations in India.
-
Question 13 of 35
13. Question
It is the largest of the seven species of sea turtles. It is found in all oceans except the Arctic and the Southern oceans. It can easily be differentiated from other modern sea turtles by its lack of a bony shell.
Based on the description given above, identify the species:
Correct
Solution (d)
Explanation:
- The Olive Ridley Sea Turtle (Lepidochelys olivacea) is the most abundant sea turtle species but is much smaller than the Leatherback, measuring up to 70 centimeters in shell length. It has a hard, bony carapace and is best known for its mass nesting events called arribadas, particularly along the Odisha coast in India. Olive Ridleys are found in the tropical regions of the Pacific, Indian, and Atlantic Oceans but do not match the description of lacking a bony shell.
- The Hawksbill Sea Turtle (Eretmochelys imbricata) is a critically endangered species known for its beautiful shell, which has a pattern of overlapping scales. It has a hard, bony shell and is smaller than the Leatherback, reaching up to 1 meter in length. Hawksbill turtles inhabit tropical reefs in the Indian, Pacific, and Atlantic Oceans. They do not lack a bony shell, so this option is incorrect.
- The Green Sea Turtle (Chelonia mydas) is named for the green color of its body fat. It has a hard, bony shell and is primarily herbivorous as an adult. Green turtles are found in tropical and subtropical seas worldwide, including the Indian Ocean. They can reachup to 1.5 meters in length but are smaller than the Leatherback. They possess a bony shell, which differentiates them from the Leatherback.
The Leatherback Sea Turtle (Dermochelys coriacea) is the largest of all living sea turtles, capable of growing up to 2 meters in length and weighing up to 900 kilograms. Unlike other sea turtles, it lacks a hard bony shell; instead, its carapace is covered by oily flesh and flexible, leathery skin, which gives the species its name. Leatherbacks have the widest global distribution of all sea turtles, inhabiting the open ocean and found in all tropical and subtropical oceans, as well as the Arctic Circle, but they are absent from the Arctic and Southern oceans. (Option (d) is correct)
Incorrect
Solution (d)
Explanation:
- The Olive Ridley Sea Turtle (Lepidochelys olivacea) is the most abundant sea turtle species but is much smaller than the Leatherback, measuring up to 70 centimeters in shell length. It has a hard, bony carapace and is best known for its mass nesting events called arribadas, particularly along the Odisha coast in India. Olive Ridleys are found in the tropical regions of the Pacific, Indian, and Atlantic Oceans but do not match the description of lacking a bony shell.
- The Hawksbill Sea Turtle (Eretmochelys imbricata) is a critically endangered species known for its beautiful shell, which has a pattern of overlapping scales. It has a hard, bony shell and is smaller than the Leatherback, reaching up to 1 meter in length. Hawksbill turtles inhabit tropical reefs in the Indian, Pacific, and Atlantic Oceans. They do not lack a bony shell, so this option is incorrect.
- The Green Sea Turtle (Chelonia mydas) is named for the green color of its body fat. It has a hard, bony shell and is primarily herbivorous as an adult. Green turtles are found in tropical and subtropical seas worldwide, including the Indian Ocean. They can reachup to 1.5 meters in length but are smaller than the Leatherback. They possess a bony shell, which differentiates them from the Leatherback.
The Leatherback Sea Turtle (Dermochelys coriacea) is the largest of all living sea turtles, capable of growing up to 2 meters in length and weighing up to 900 kilograms. Unlike other sea turtles, it lacks a hard bony shell; instead, its carapace is covered by oily flesh and flexible, leathery skin, which gives the species its name. Leatherbacks have the widest global distribution of all sea turtles, inhabiting the open ocean and found in all tropical and subtropical oceans, as well as the Arctic Circle, but they are absent from the Arctic and Southern oceans. (Option (d) is correct)
-
Question 14 of 35
14. Question
Which of the following is not a mammal?
Correct
Solution (c)
Explanation:
- The Dugong (Dugong dugon), also known as the “sea cow,” is a marine mammal found in warm coastal waters, including the Gulf of Mannar and the Andaman and Nicobar Islands in India. It is herbivorous, feeding mainly on seagrass.
- The Indian Pangolin (Manis crassicaudata) is a mammal covered in protective keratin scales. It is found throughout the Indian subcontinent and is known for its ant and termite diet. When threatened, it rolls into a ball, using its scales as armor.
- The King Cobra (Ophiophagus hannah) is the world’s longest venomous snake and is a reptile, not a mammal. It is found predominantly in forests in India and Southeast Asia. The King Cobra is known for its potent venom and distinctive hood. (Option (c) is correct)
- The Flying Fox refers to large bats of the genus Pteropus. They are mammals found across India and play a crucial role in pollination and seed dispersal. Despite their name, they are not foxes but are so named due to their fox-like faces.
Incorrect
Solution (c)
Explanation:
- The Dugong (Dugong dugon), also known as the “sea cow,” is a marine mammal found in warm coastal waters, including the Gulf of Mannar and the Andaman and Nicobar Islands in India. It is herbivorous, feeding mainly on seagrass.
- The Indian Pangolin (Manis crassicaudata) is a mammal covered in protective keratin scales. It is found throughout the Indian subcontinent and is known for its ant and termite diet. When threatened, it rolls into a ball, using its scales as armor.
- The King Cobra (Ophiophagus hannah) is the world’s longest venomous snake and is a reptile, not a mammal. It is found predominantly in forests in India and Southeast Asia. The King Cobra is known for its potent venom and distinctive hood. (Option (c) is correct)
- The Flying Fox refers to large bats of the genus Pteropus. They are mammals found across India and play a crucial role in pollination and seed dispersal. Despite their name, they are not foxes but are so named due to their fox-like faces.
-
Question 15 of 35
15. Question
Which of the following animal is the world’s most endangered marine mammal and is also known as the ‘panda of the sea’?
Correct
Solution (a)
Explanation:
- The Vaquita Porpoise (Phocoena sinus) is the world’s most endangered marine mammal, with an estimated population of fewer than 10 individuals as of recent assessments. It is endemic to the northern Gulf of California, Mexico.
- The Vaquita Porpoise is often called the ‘panda of the sea’ due to the distinctive dark rings around its eyes and mouth, resembling the markings of a panda. The primary threat to the Vaquita is accidental entanglement in illegal gillnets used for fishing the totoaba, a fish whose swim bladder is highly valued in traditional medicine. (Option (a) is correct)
- The Dugong (Dugong dugon), also known as the ‘sea cow’, is a marine mammal found in warm coastal waters from East Africa to Australia, including the waters around India. It is the only herbivorous marine mammal and is classified as Vulnerable due to habitat loss, hunting, and entanglement in fishing gear.
- The Irrawaddy Dolphin (Orcaella brevirostris) is found in coastal areas, estuaries, and rivers in South and Southeast Asia, including parts of India such as the Chilika Lake in Odisha. It is classified as Endangered due to habitat loss, pollution, and fishing net entanglements.
- The Finless Porpoise (Neophocaena phocaenoides) inhabits coastal waters of Asia, including Indian waters along the eastern coast. It is classified as Vulnerable due to threats like habitat degradation, pollution, and incidental capture in fishing gear.
Incorrect
Solution (a)
Explanation:
- The Vaquita Porpoise (Phocoena sinus) is the world’s most endangered marine mammal, with an estimated population of fewer than 10 individuals as of recent assessments. It is endemic to the northern Gulf of California, Mexico.
- The Vaquita Porpoise is often called the ‘panda of the sea’ due to the distinctive dark rings around its eyes and mouth, resembling the markings of a panda. The primary threat to the Vaquita is accidental entanglement in illegal gillnets used for fishing the totoaba, a fish whose swim bladder is highly valued in traditional medicine. (Option (a) is correct)
- The Dugong (Dugong dugon), also known as the ‘sea cow’, is a marine mammal found in warm coastal waters from East Africa to Australia, including the waters around India. It is the only herbivorous marine mammal and is classified as Vulnerable due to habitat loss, hunting, and entanglement in fishing gear.
- The Irrawaddy Dolphin (Orcaella brevirostris) is found in coastal areas, estuaries, and rivers in South and Southeast Asia, including parts of India such as the Chilika Lake in Odisha. It is classified as Endangered due to habitat loss, pollution, and fishing net entanglements.
- The Finless Porpoise (Neophocaena phocaenoides) inhabits coastal waters of Asia, including Indian waters along the eastern coast. It is classified as Vulnerable due to threats like habitat degradation, pollution, and incidental capture in fishing gear.
-
Question 16 of 35
16. Question
India’s first wildlife conservation reserve dedicated exclusively for blackbuck is located in the state of?
Correct
Solution (c)
Explanation:
- Rajasthan is home to the Tal Chhapar Wildlife Sanctuary, which is known for its significant population of blackbucks.
- Madhya Pradesh has rich biodiversity and habitats supporting blackbuck populations, such as the Kanha National Park.
- The Uttar Pradesh government has approved India’s first wildlife conservation reserve dedicated exclusively for blackbuck in the Meja Forest Division of Prayagraj (formerly Allahabad) in 2018. The reserve is located in the trans-Yamuna region near Prayagraj. This initiative aims to protect the blackbuck population by providing a safe habitat and promoting conservation efforts. (Option (c) is correct)
- Haryana has taken steps to conserve blackbucks and, in 2021, notified the setting up of a blackbuck conservation reserve in the Bhiwani district. However, this was after Uttar Pradesh’s initiative.
Incorrect
Solution (c)
Explanation:
- Rajasthan is home to the Tal Chhapar Wildlife Sanctuary, which is known for its significant population of blackbucks.
- Madhya Pradesh has rich biodiversity and habitats supporting blackbuck populations, such as the Kanha National Park.
- The Uttar Pradesh government has approved India’s first wildlife conservation reserve dedicated exclusively for blackbuck in the Meja Forest Division of Prayagraj (formerly Allahabad) in 2018. The reserve is located in the trans-Yamuna region near Prayagraj. This initiative aims to protect the blackbuck population by providing a safe habitat and promoting conservation efforts. (Option (c) is correct)
- Haryana has taken steps to conserve blackbucks and, in 2021, notified the setting up of a blackbuck conservation reserve in the Bhiwani district. However, this was after Uttar Pradesh’s initiative.
-
Question 17 of 35
17. Question
Consider the following pairs:
Animal State animal of
- Sangai – Manipur
- Gaur – Goa
- Sambar – Tamil Nadu
Which of the pairs given above is/are correctly matched?
Correct
Solution (a)
Explanation:
- The Sangai (Rucervus eldii eldii), also known as the brow-antlered deer or dancing deer, is the state animal of Manipur. It is an endangered subspecies found only in Manipur, particularly in the Keibul Lamjao National Park, which is the world’s only floating national park located on the Loktak Lake. The Sangai is significant to the cultural heritage of Manipur and is an integral part of the state’s identity. (Pair 1 is correct)
- The Gaur (Bos gaurus), commonly known as the Indian Bison, is the state animal of Goa. It is the largest extant bovine species and is native to South Asia and Southeast Asia. In Goa, Gaurs are found in the Bhagwan Mahavir Wildlife Sanctuary and other forested areas. The animal holds ecological significance due to its role in the forest ecosystem. (Pair 2 is correct)
- The Sambar Deer (Rusa unicolor) is a large deer native to the Indian subcontinent and is found in various parts of India, including Tamil Nadu. However, it is not the state animal of Tamil Nadu. The state animal of Tamil Nadu is the Nilgiri Tahr (Nilgiritragus hylocrius), an endangered mountain ungulate endemic to the Nilgiri Hills and the southern portion of the Western Ghats within Tamil Nadu and Kerala. The Nilgiri Tahr is known for its stocky build and curved horns and is adapted to the montane grassland ecosystem. (Pair 3 is incorrect)
Incorrect
Solution (a)
Explanation:
- The Sangai (Rucervus eldii eldii), also known as the brow-antlered deer or dancing deer, is the state animal of Manipur. It is an endangered subspecies found only in Manipur, particularly in the Keibul Lamjao National Park, which is the world’s only floating national park located on the Loktak Lake. The Sangai is significant to the cultural heritage of Manipur and is an integral part of the state’s identity. (Pair 1 is correct)
- The Gaur (Bos gaurus), commonly known as the Indian Bison, is the state animal of Goa. It is the largest extant bovine species and is native to South Asia and Southeast Asia. In Goa, Gaurs are found in the Bhagwan Mahavir Wildlife Sanctuary and other forested areas. The animal holds ecological significance due to its role in the forest ecosystem. (Pair 2 is correct)
- The Sambar Deer (Rusa unicolor) is a large deer native to the Indian subcontinent and is found in various parts of India, including Tamil Nadu. However, it is not the state animal of Tamil Nadu. The state animal of Tamil Nadu is the Nilgiri Tahr (Nilgiritragus hylocrius), an endangered mountain ungulate endemic to the Nilgiri Hills and the southern portion of the Western Ghats within Tamil Nadu and Kerala. The Nilgiri Tahr is known for its stocky build and curved horns and is adapted to the montane grassland ecosystem. (Pair 3 is incorrect)
-
Question 18 of 35
18. Question
Ameenpur Lake is the first Biodiversity Heritage Site in the country. It is located in an urban area. Identify the city in which it is located?
Correct
Solution (d)
Explanation:
- Kolkata is the capital city of the Indian state of West Bengal, situated on the eastern bank of the Hooghly River. While Kolkata has several notable water bodies such as Rabindra Sarobar and Subhas Sarobar, Ameenpur Lake is not among them.
- Chennai is the capital city of the Indian state of Tamil Nadu, located on the Coromandel Coast off the Bay of Bengal. Chennai is known for water bodies like Pulicat Lake and Chembarambakkam Lake, but Ameenpur Lake is not situated in this region.
- Bengaluru is the capital city of the Indian state of Karnataka. The city is known for its numerous lakes, such as Ulsoor Lake, Hebbal Lake, and Madiwala Lake, but Ameenpur Lake is not among them.
- Ameenpur Lake is located on the outskirts of Hyderabad, the capital city of the Indian state of Telangana. In November 2016, Ameenpur Lake became the first water body in India to be declared a Biodiversity Heritage Site under the Biological Diversity Act, 2002. The lake is renowned for its rich biodiversity, particularly bird species, attracting bird watchers and nature enthusiasts. (Option (d) is correct)
Incorrect
Solution (d)
Explanation:
- Kolkata is the capital city of the Indian state of West Bengal, situated on the eastern bank of the Hooghly River. While Kolkata has several notable water bodies such as Rabindra Sarobar and Subhas Sarobar, Ameenpur Lake is not among them.
- Chennai is the capital city of the Indian state of Tamil Nadu, located on the Coromandel Coast off the Bay of Bengal. Chennai is known for water bodies like Pulicat Lake and Chembarambakkam Lake, but Ameenpur Lake is not situated in this region.
- Bengaluru is the capital city of the Indian state of Karnataka. The city is known for its numerous lakes, such as Ulsoor Lake, Hebbal Lake, and Madiwala Lake, but Ameenpur Lake is not among them.
- Ameenpur Lake is located on the outskirts of Hyderabad, the capital city of the Indian state of Telangana. In November 2016, Ameenpur Lake became the first water body in India to be declared a Biodiversity Heritage Site under the Biological Diversity Act, 2002. The lake is renowned for its rich biodiversity, particularly bird species, attracting bird watchers and nature enthusiasts. (Option (d) is correct)
-
Question 19 of 35
19. Question
Recently, there has been a growing awareness in our country about the importance of water hyacinth because it is found to be a sustainable source of:
Correct
Solution (c)
Explanation:
- Water hyacinth is an aquatic plant and does not contain the minerals or materials required for steel production. Steel is an alloy primarily made from iron ore, carbon, and other elements. Water hyacinth cannot be used in any process related to the production of steel.
- Natural rubber is extracted from the latex of certain trees, primarily the rubber tree (Hevea brasiliensis). Water hyacinth does not produce latex or any substance that can be used to extract natural rubber. Therefore, it is not a source of natural rubber.
- Water hyacinth is an invasive aquatic plant that poses environmental challenges by clogging waterways. Recently, there has been a growing awareness and initiative, especially in Assam and other Northeastern states of India, to utilize water hyacinth as a sustainable raw material for making handicrafts. Artisans weave the dried stems into products like baskets, bags, mats, and furniture. This not only provides livelihood opportunities for local communities, particularly women, but also helps in environmental management by controlling the spread of this invasive species. (Option (c) is correct)
- While some plants are used in traditional medicine for treating diabetes, water hyacinth is not recognized as a source of medicinal compounds for this purpose in India. There is no substantial scientific evidence or official endorsement supporting the use of water hyacinth in diabetes treatment.
Incorrect
Solution (c)
Explanation:
- Water hyacinth is an aquatic plant and does not contain the minerals or materials required for steel production. Steel is an alloy primarily made from iron ore, carbon, and other elements. Water hyacinth cannot be used in any process related to the production of steel.
- Natural rubber is extracted from the latex of certain trees, primarily the rubber tree (Hevea brasiliensis). Water hyacinth does not produce latex or any substance that can be used to extract natural rubber. Therefore, it is not a source of natural rubber.
- Water hyacinth is an invasive aquatic plant that poses environmental challenges by clogging waterways. Recently, there has been a growing awareness and initiative, especially in Assam and other Northeastern states of India, to utilize water hyacinth as a sustainable raw material for making handicrafts. Artisans weave the dried stems into products like baskets, bags, mats, and furniture. This not only provides livelihood opportunities for local communities, particularly women, but also helps in environmental management by controlling the spread of this invasive species. (Option (c) is correct)
- While some plants are used in traditional medicine for treating diabetes, water hyacinth is not recognized as a source of medicinal compounds for this purpose in India. There is no substantial scientific evidence or official endorsement supporting the use of water hyacinth in diabetes treatment.
-
Question 20 of 35
20. Question
From an ecological point of view, which one of the following is significant for being
the only floating national park in the world, providing habitat to unique flora and fauna?
Correct
Solution (b)
Explanation:
-
- Keoladeo National Park, also known as Bharatpur Bird Sanctuary, is located in Rajasthan. It is a UNESCO World Heritage Site renowned for its rich avian diversity, especially migratory birds during the winter season. While it is an important wetland reserve, it is not a floating national park.
- Keibul Lamjao National Park is located in the Bishnupur district of Manipur and is the only floating national park in the world. It is situated on the southern shores of Loktak Lake, which features floating decomposed plant materials called ‘Phumdis’. The park is the natural habitat of the endangered Sangai deer (Rucervus eldii eldii), also known as the Manipur brow-antlered deer.
- Unique Features:
-
- Floating Vegetation: The park’s terrain consists of floating masses of vegetation that support diverse flora and fauna.
- Endemic Species: Home to the Sangai deer, which is endemic to Manipur. (Option (b) is correct)
- Kaziranga National Park is located in Assam and is famous for hosting two-thirds of the world’s Great One-horned Rhinoceroses. It is a UNESCO World Heritage Site known for its high species diversity and conservation efforts. However, it is not a floating national park.
- Silent Valley National Park is located in the Nilgiri Hills of Kerala. It is known for its unique and undisturbed tropical evergreen rainforest and rich biodiversity, including the endangered Lion-tailed Macaque. Despite its ecological importance, it is not a floating national park.
Incorrect
Solution (b)
Explanation:
-
- Keoladeo National Park, also known as Bharatpur Bird Sanctuary, is located in Rajasthan. It is a UNESCO World Heritage Site renowned for its rich avian diversity, especially migratory birds during the winter season. While it is an important wetland reserve, it is not a floating national park.
- Keibul Lamjao National Park is located in the Bishnupur district of Manipur and is the only floating national park in the world. It is situated on the southern shores of Loktak Lake, which features floating decomposed plant materials called ‘Phumdis’. The park is the natural habitat of the endangered Sangai deer (Rucervus eldii eldii), also known as the Manipur brow-antlered deer.
- Unique Features:
-
- Floating Vegetation: The park’s terrain consists of floating masses of vegetation that support diverse flora and fauna.
- Endemic Species: Home to the Sangai deer, which is endemic to Manipur. (Option (b) is correct)
- Kaziranga National Park is located in Assam and is famous for hosting two-thirds of the world’s Great One-horned Rhinoceroses. It is a UNESCO World Heritage Site known for its high species diversity and conservation efforts. However, it is not a floating national park.
- Silent Valley National Park is located in the Nilgiri Hills of Kerala. It is known for its unique and undisturbed tropical evergreen rainforest and rich biodiversity, including the endangered Lion-tailed Macaque. Despite its ecological importance, it is not a floating national park.
-
Question 21 of 35
21. Question
With regard to the PM Internship Scheme, consider the following statements:
- It aims to bridge the gap between academic learning and real-world experience by offering internships across the top 500 companies.
- It provides interns with monthly financial assistance of Rs 15,000 for 12 months and a one-time grant of Rs 6,000.
- It is not open to post-graduates and individuals from families with government jobs.
Select the correct answer using the codes below:
Correct
Solution (c)
Statement Analysis
Context: The Union Corporate Affairs Ministry said that the pilot scheme of the Prime Minister’s Internship Scheme received approximately 6.21 lakh applications against 1.27 lakh opportunities.
- The PM Internship Scheme aims to bridge the gap between academic learning and real-world experience by offering internships across the top 500 companies. Hence statement 1 is correct.
- It will help youth to gain exposure to real-life business environments, across varied professions and employment opportunities.
- It provides interns with monthly financial assistance of Rs 5,000 for 12 months and a one-time grant of Rs 6,000. Hence statement 2 is incorrect.
- The pilot phase targets 1.25 lakh youth, with a five-year goal to facilitate internships for one crore young individuals.
- It is not open to post-graduates and individuals from families with government jobs. Hence statement 3 is correct.
- A candidate who graduated from premier institutes such as IIT, IIM, or IISER, and those who have CA, or CMA qualifications would not be eligible to apply for this internship.
- Anyone from a household that includes a person who earned an income of Rs.8 lakh or more in 2023-24, will not be eligible.
- Eligible for the scheme:
- Candidates aged between 21 and 24 years and who are not engaged in full-time employment.
- Internships are available to those who have passed class 10 or higher.
Source: https://www.thehindu.com/news/national/621-lakh-applications-for-127-lakh-opportunities-under-pm-internship-scheme/article69040726.ece
Incorrect
Solution (c)
Statement Analysis
Context: The Union Corporate Affairs Ministry said that the pilot scheme of the Prime Minister’s Internship Scheme received approximately 6.21 lakh applications against 1.27 lakh opportunities.
- The PM Internship Scheme aims to bridge the gap between academic learning and real-world experience by offering internships across the top 500 companies. Hence statement 1 is correct.
- It will help youth to gain exposure to real-life business environments, across varied professions and employment opportunities.
- It provides interns with monthly financial assistance of Rs 5,000 for 12 months and a one-time grant of Rs 6,000. Hence statement 2 is incorrect.
- The pilot phase targets 1.25 lakh youth, with a five-year goal to facilitate internships for one crore young individuals.
- It is not open to post-graduates and individuals from families with government jobs. Hence statement 3 is correct.
- A candidate who graduated from premier institutes such as IIT, IIM, or IISER, and those who have CA, or CMA qualifications would not be eligible to apply for this internship.
- Anyone from a household that includes a person who earned an income of Rs.8 lakh or more in 2023-24, will not be eligible.
- Eligible for the scheme:
- Candidates aged between 21 and 24 years and who are not engaged in full-time employment.
- Internships are available to those who have passed class 10 or higher.
Source: https://www.thehindu.com/news/national/621-lakh-applications-for-127-lakh-opportunities-under-pm-internship-scheme/article69040726.ece
-
Question 22 of 35
22. Question
Recently in the news, Rooppur Nuclear Power Plant is located in which of the following country?
Correct
Solution (b)
Statement Analysis
Context: An anti-corruption outfit in Bangladesh has initiated an investigation into the $12.65 billion Rooppur nuclear powerplant that is being constructed with Russian assistance.
- Recently in the news, Rooppur Nuclear Power Plant is located in Bangladesh. Hence option b is correct.
- It is a nuclear power plant under construction in Iswardi, Pabna District, Rajshahi Division, Bangladesh.
- It will be Bangladesh’s first source of nuclear power.
- Bangladesh Atomic Energy Commission (BAEC) is the owner of the Rooppur nuclear plant.
- It features two VVER-1200 reactors, created by the Russian company Rosatom.
- The construction is largely funded by a loan from Russia, to be paid back over 20 years.
Source: https://www.thehindu.com/news/international/bangladesh-initiates-investigation-into-russia-backed-rooppur-nuclear-project/article69018424.ece
Incorrect
Solution (b)
Statement Analysis
Context: An anti-corruption outfit in Bangladesh has initiated an investigation into the $12.65 billion Rooppur nuclear powerplant that is being constructed with Russian assistance.
- Recently in the news, Rooppur Nuclear Power Plant is located in Bangladesh. Hence option b is correct.
- It is a nuclear power plant under construction in Iswardi, Pabna District, Rajshahi Division, Bangladesh.
- It will be Bangladesh’s first source of nuclear power.
- Bangladesh Atomic Energy Commission (BAEC) is the owner of the Rooppur nuclear plant.
- It features two VVER-1200 reactors, created by the Russian company Rosatom.
- The construction is largely funded by a loan from Russia, to be paid back over 20 years.
Source: https://www.thehindu.com/news/international/bangladesh-initiates-investigation-into-russia-backed-rooppur-nuclear-project/article69018424.ece
-
Question 23 of 35
23. Question
Exercise Surya Kiran is a joint military exercise between which of the following countries?
Correct
Solution (d)
Statement Analysis
Context: The Indian Army contingent comprising 334 personnel departed for Nepal to participate in the 18th edition of Battalion Level Joint Military Exercise SURYA KIRAN.
- Exercise Surya Kiran is a joint military exercise between India and Nepal. Hence option d is correct.
- It is an annual event conducted alternately in both countries.
- It aims to enhance interoperability in jungle warfare, counter-terrorism operations in mountains, and Humanitarian Assistance and Disaster Relief under the United Nations Charter.
Source: https://pib.gov.in/PressReleasePage.aspx?PRID=2088529
Incorrect
Solution (d)
Statement Analysis
Context: The Indian Army contingent comprising 334 personnel departed for Nepal to participate in the 18th edition of Battalion Level Joint Military Exercise SURYA KIRAN.
- Exercise Surya Kiran is a joint military exercise between India and Nepal. Hence option d is correct.
- It is an annual event conducted alternately in both countries.
- It aims to enhance interoperability in jungle warfare, counter-terrorism operations in mountains, and Humanitarian Assistance and Disaster Relief under the United Nations Charter.
Source: https://pib.gov.in/PressReleasePage.aspx?PRID=2088529
-
Question 24 of 35
24. Question
Consider the following statements about GPS Spoofing:
- It refers to the practice of manipulating a GPS receiver by broadcasting false GPS signals.
- It exploits the inherent vulnerabilities in the GPS infrastructure – the weak signal strength of GPS satellites.
Which of the above statement/s is/are correct?
Correct
Solution (c)
Statement Analysis
Context: Instances of GPS interference on passenger aircraft, including ‘spoofing’ with false signals, are on the rise over conflict zones globally, including on India’s borders with Pakistan, which are among the top sites for such occurrences, according to OPSGROUP, a voluntary group of 8,000 aviation personnel, including pilots, who share information on risks to flights.
- GPS Spoofing refers to the practice of manipulating a GPS receiver by broadcasting false GPS signals. Hence statement 1 is correct.
- It misleads the GPS receiver into believing it is located somewhere it is not, resulting in the device providing inaccurate location data.
- It undermines the reliability of GPS data, which is vital for a variety of applications, from navigation to time synchronization and more.
- It exploits the inherent vulnerabilities in the GPS infrastructure – the weak signal strength of GPS satellites. Hence statement 2 is correct.
Source: https://www.thehindu.com/news/national/frequent-gps-interference-including-spoofing-near-indias-border-with-pakistan-myanmar/article69037483.ece
Incorrect
Solution (c)
Statement Analysis
Context: Instances of GPS interference on passenger aircraft, including ‘spoofing’ with false signals, are on the rise over conflict zones globally, including on India’s borders with Pakistan, which are among the top sites for such occurrences, according to OPSGROUP, a voluntary group of 8,000 aviation personnel, including pilots, who share information on risks to flights.
- GPS Spoofing refers to the practice of manipulating a GPS receiver by broadcasting false GPS signals. Hence statement 1 is correct.
- It misleads the GPS receiver into believing it is located somewhere it is not, resulting in the device providing inaccurate location data.
- It undermines the reliability of GPS data, which is vital for a variety of applications, from navigation to time synchronization and more.
- It exploits the inherent vulnerabilities in the GPS infrastructure – the weak signal strength of GPS satellites. Hence statement 2 is correct.
Source: https://www.thehindu.com/news/national/frequent-gps-interference-including-spoofing-near-indias-border-with-pakistan-myanmar/article69037483.ece
-
Question 25 of 35
25. Question
In the context of the Indian economy, consider the following statements:
- Nominal Effective Exchange Rate is the weighted average of a currency’s bilateral exchange rates with multiple trading partner currencies.
- Real Effective Exchange Rate adjusts for relative price levels between domestic and foreign economies.
Which of the above statement/s is/are correct?
Correct
Solution (c)
Statement Analysis
Context: The rupee is hitting fresh lows against the US dollar each day, yet its exchange rate has scaled an all-time high in “real effective” terms.
- Nominal Effective Exchange Rate is the weighted average of a currency’s bilateral exchange rates with multiple trading partner currencies. Hence statement 1 is correct.
- It reflects the nominal strength of a currency but does not adjust for inflation or price level differences.
- A rise in NEER indicates nominal appreciation, while a decline signals depreciation.
- Real Effective Exchange Rate adjusts for relative price levels between domestic and foreign economies. Hence statement 2 is correct.
- It is a purchasing power parity (PPP)-adjusted measure of competitiveness.
Source: https://indianexpress.com/article/business/economy/as-rupee-hits-new-lows-real-effective-exchange-rate-soars-to-an-all-time-high-9744407/
Incorrect
Solution (c)
Statement Analysis
Context: The rupee is hitting fresh lows against the US dollar each day, yet its exchange rate has scaled an all-time high in “real effective” terms.
- Nominal Effective Exchange Rate is the weighted average of a currency’s bilateral exchange rates with multiple trading partner currencies. Hence statement 1 is correct.
- It reflects the nominal strength of a currency but does not adjust for inflation or price level differences.
- A rise in NEER indicates nominal appreciation, while a decline signals depreciation.
- Real Effective Exchange Rate adjusts for relative price levels between domestic and foreign economies. Hence statement 2 is correct.
- It is a purchasing power parity (PPP)-adjusted measure of competitiveness.
Source: https://indianexpress.com/article/business/economy/as-rupee-hits-new-lows-real-effective-exchange-rate-soars-to-an-all-time-high-9744407/
-
Question 26 of 35
26. Question
The Durand Line was demarcated as the border between:
Correct
Solution (b)
Statement Analysis
Context: The clashes between the two countries on Pakistan’s western border — Afghanistan retaliated with strikes across the Durand Line after Pakistan targeted what it called hideouts for militants in the former — mark a deterioration of ties and the deepening of instability in an already volatile region.
- The Durand Line was demarcated as the border between British India and Afghanistan. Hence option b is correct.
- The Durand Line is a legacy of the 19th-century Great Game between the Russian and British empires in which Afghanistan was used as a buffer by the British against feared Russian expansionism to its east.
- The agreement demarcating what became known as the Durand Line was signed on November 12, 1893, between the British civil servant Henry Mortimer Durand and Amir Abdur Rahman, then the Afghan ruler.
Source: https://indianexpress.com/article/opinion/editorials/express-view-a-crisis-to-the-west-9750595/?ref=view_opinion
Incorrect
Solution (b)
Statement Analysis
Context: The clashes between the two countries on Pakistan’s western border — Afghanistan retaliated with strikes across the Durand Line after Pakistan targeted what it called hideouts for militants in the former — mark a deterioration of ties and the deepening of instability in an already volatile region.
- The Durand Line was demarcated as the border between British India and Afghanistan. Hence option b is correct.
- The Durand Line is a legacy of the 19th-century Great Game between the Russian and British empires in which Afghanistan was used as a buffer by the British against feared Russian expansionism to its east.
- The agreement demarcating what became known as the Durand Line was signed on November 12, 1893, between the British civil servant Henry Mortimer Durand and Amir Abdur Rahman, then the Afghan ruler.
Source: https://indianexpress.com/article/opinion/editorials/express-view-a-crisis-to-the-west-9750595/?ref=view_opinion
-
Question 27 of 35
27. Question
Consider the following statements about Lion-tailed Macaque:
- It is endemic to evergreen rainforests of the southern part of Eastern Ghats.
- It is an arboreal, diurnal, and territorial creature.
- It is classified as endangered on the IUCN Red List and protected under Appendix I of CITES.
How many of the above statement/s is/are correct?
Correct
Solution (b)
Statement Analysis
Context: A recent study warns of a growing threat to the critically endangered lion-tailed macaque, endemic to the Western Ghats of India due to increasing human interaction.
- Lion-tailed Macaque is endemic to evergreen rainforests of the southern part of the Western Ghats. Hence statement 1 is incorrect.
- One of the distinguishing features of this species is that males define the boundaries of their home ranges by calls.
- It is omnivorous and feeds upon a wide variety of food, although fruits form the major part of their diet.
- It is an arboreal, diurnal, and territorial creature. Hence statement 2 is correct.
- It is named due to its lion-like, long, thin, and tufted tail. Also, they are characterized by the grey mane around their face.
- In the meantime, this animal is one of the smallest macaque species in the world.
- It is classified as endangered on the IUCN Red List and protected under Appendix I of CITES. Hence statement 3 is correct.
Source: https://www.thehindu.com/sci-tech/energy-and-environment/human-interactions-threaten-endangered-lion-tailed-macaque/article69022248.ece
Incorrect
Solution (b)
Statement Analysis
Context: A recent study warns of a growing threat to the critically endangered lion-tailed macaque, endemic to the Western Ghats of India due to increasing human interaction.
- Lion-tailed Macaque is endemic to evergreen rainforests of the southern part of the Western Ghats. Hence statement 1 is incorrect.
- One of the distinguishing features of this species is that males define the boundaries of their home ranges by calls.
- It is omnivorous and feeds upon a wide variety of food, although fruits form the major part of their diet.
- It is an arboreal, diurnal, and territorial creature. Hence statement 2 is correct.
- It is named due to its lion-like, long, thin, and tufted tail. Also, they are characterized by the grey mane around their face.
- In the meantime, this animal is one of the smallest macaque species in the world.
- It is classified as endangered on the IUCN Red List and protected under Appendix I of CITES. Hence statement 3 is correct.
Source: https://www.thehindu.com/sci-tech/energy-and-environment/human-interactions-threaten-endangered-lion-tailed-macaque/article69022248.ece
-
Question 28 of 35
28. Question
With the present state of development, Artificial Intelligence can effectively do which of the following?
- Bring down electricity consumption in industrial units
- Create meaningful short stories and songs
- Disease diagnosis
- Text-to-Speech Conversion
- Wireless transmission of electrical energy
Select the correct answer using the code given below:
Correct
Solution (d)
Statement Analysis
Context: 2024 has been a year of significant advancements, with AI cementing its role as the defining technology of the decade. While many predictions came true, the speed and depth of some changes — especially in responsible AI and greener tech — highlighted both the promise and challenges of this transformative year.
Artificial Intelligence has various applications in today’s society in multiple industries, such as healthcare, entertainment, finance, education, etc.
- Some high-profile applications are: advanced web search engines (e.g., Google Search), recommendation systems (used by YouTube, Amazon, and Netflix), understanding human speech (such as Google Assistant, Siri, and Alexa), self-driving cars (e.g., Waymo), generative and creative tools (ChatGPT and AI art), and superhuman play and analysis in strategy games (such as chess and Go).
- AI has been used in disease diagnosis, creating songs like ‘I am AI’ and ‘Daddy’s Car’ and creating short stories and fiction. AI has been used in Text-to-speech conversion, e.g. Cerewave AI.
- Artificial Intelligence has also found use in the power industry, e.g. Machine-learning assisted power transfer (based on AI) using magnetic resonance and AI used for energy efficiency.
Hence option d is correct.
Source: https://indianexpress.com/article/explained/explained-sci-tech/looking-at-2025-tech-rising-above-the-ai-line-9750694/
Incorrect
Solution (d)
Statement Analysis
Context: 2024 has been a year of significant advancements, with AI cementing its role as the defining technology of the decade. While many predictions came true, the speed and depth of some changes — especially in responsible AI and greener tech — highlighted both the promise and challenges of this transformative year.
Artificial Intelligence has various applications in today’s society in multiple industries, such as healthcare, entertainment, finance, education, etc.
- Some high-profile applications are: advanced web search engines (e.g., Google Search), recommendation systems (used by YouTube, Amazon, and Netflix), understanding human speech (such as Google Assistant, Siri, and Alexa), self-driving cars (e.g., Waymo), generative and creative tools (ChatGPT and AI art), and superhuman play and analysis in strategy games (such as chess and Go).
- AI has been used in disease diagnosis, creating songs like ‘I am AI’ and ‘Daddy’s Car’ and creating short stories and fiction. AI has been used in Text-to-speech conversion, e.g. Cerewave AI.
- Artificial Intelligence has also found use in the power industry, e.g. Machine-learning assisted power transfer (based on AI) using magnetic resonance and AI used for energy efficiency.
Hence option d is correct.
Source: https://indianexpress.com/article/explained/explained-sci-tech/looking-at-2025-tech-rising-above-the-ai-line-9750694/
-
Question 29 of 35
29. Question
Consider the following actions which the Government can take:
- Devaluing the domestic currency.
- Reduction in the export subsidy.
- Adopting suitable policies that attract greater FDI and more funds from FIIs.
Which of the above action/actions can help in reducing the current account deficit?
Correct
Solution (d)
Statement Analysis
Context: The country’s current account deficit (CAD) moderated marginally to $11.2 billion, or 1.2 percent of gross domestic product (GDP), in July-September 2024 quarter from $11.3 billion, or 1.3 percent of GDP, in the same period of the previous fiscal.
- CAD exists due to a host of factors including existing exchange rate, consumer spending level, capital inflow, inflation level, and prevailing interest rate.
- For the Current Account Deficit in India, crude oil and gold imports are the primary reasons behind the high CAD.
- Devaluing the domestic currency makes the export of a country competitive in the international market, which eventually helps to increase exports and earn foreign currency. This can help to reduce CAD. Hence statement 1 is correct.
- Reduction in the export subsidy will discourage export and exports from India will become expensive at the international market vis-a-vis other countries. Thus, CAD will increase. Hence statement 2 is incorrect.
- The Current Account Deficit could be reduced by boosting exports and curbing non-essential imports such as gold, mobiles, and electronics. Hence statement 3 is correct.
Source: https://indianexpress.com/article/business/current-account-deficit-narrows-to-11-2-bn-in-q2-rbi-9748003/
Incorrect
Solution (d)
Statement Analysis
Context: The country’s current account deficit (CAD) moderated marginally to $11.2 billion, or 1.2 percent of gross domestic product (GDP), in July-September 2024 quarter from $11.3 billion, or 1.3 percent of GDP, in the same period of the previous fiscal.
- CAD exists due to a host of factors including existing exchange rate, consumer spending level, capital inflow, inflation level, and prevailing interest rate.
- For the Current Account Deficit in India, crude oil and gold imports are the primary reasons behind the high CAD.
- Devaluing the domestic currency makes the export of a country competitive in the international market, which eventually helps to increase exports and earn foreign currency. This can help to reduce CAD. Hence statement 1 is correct.
- Reduction in the export subsidy will discourage export and exports from India will become expensive at the international market vis-a-vis other countries. Thus, CAD will increase. Hence statement 2 is incorrect.
- The Current Account Deficit could be reduced by boosting exports and curbing non-essential imports such as gold, mobiles, and electronics. Hence statement 3 is correct.
Source: https://indianexpress.com/article/business/current-account-deficit-narrows-to-11-2-bn-in-q2-rbi-9748003/
-
Question 30 of 35
30. Question
With reference to Greenland, consider the following statements:
- It is located between the continents of North America and Europe in the North Atlantic Ocean.
- It is now an autonomous province of Denmark.
- Nuuk is the capital of Greenland.
How many of the above statement/s is/are correct?
Correct
Solution (c)
Statement Analysis
Context: US President-elect Donald Trump has once again expressed interest in buying Greenland, and Greenland has again stated it is not for sale.
- Greenland is located between the continents of North America and Europe in the North Atlantic Ocean. Hence statement 1 is correct.
- It is geographically considered a part of the North American continent.
- It is surrounded by the Arctic Ocean to the north; by the Greenland Sea to the east; by the North Atlantic Ocean to the southeast; Davis Strait to the southwest and Baffin Bay to the west.
- It is now an autonomous province of Denmark. Hence statement 2 is correct.
- It is in the polar zone, where winter temperatures reach as low as -50°C and summer temperatures rarely exceed 10–15°C.
- Nuuk is the capital of Greenland. Hence statement 3 is correct.
Source: https://indianexpress.com/article/explained/explained-global/why-trump-wants-to-buy-greenland-past-attempt-9742583/
Incorrect
Solution (c)
Statement Analysis
Context: US President-elect Donald Trump has once again expressed interest in buying Greenland, and Greenland has again stated it is not for sale.
- Greenland is located between the continents of North America and Europe in the North Atlantic Ocean. Hence statement 1 is correct.
- It is geographically considered a part of the North American continent.
- It is surrounded by the Arctic Ocean to the north; by the Greenland Sea to the east; by the North Atlantic Ocean to the southeast; Davis Strait to the southwest and Baffin Bay to the west.
- It is now an autonomous province of Denmark. Hence statement 2 is correct.
- It is in the polar zone, where winter temperatures reach as low as -50°C and summer temperatures rarely exceed 10–15°C.
- Nuuk is the capital of Greenland. Hence statement 3 is correct.
Source: https://indianexpress.com/article/explained/explained-global/why-trump-wants-to-buy-greenland-past-attempt-9742583/
-
Question 31 of 35
31. Question
Passage – 1
Most of our parents would probably say we would be far better off if we could use the lessons they learned the hard way – lessons that would have saved them time, effort and maybe even heartbreak had they learnt them sooner. But did they live life based on the learnings of their own parents’ mistakes before us? Do we learn our lessons from history as individuals, as nations, as mankind? As human beings, we go in cycles from one milestone to the next. Starting over is at the core of our existence. Even the 50 to 75 trillion cells that make up a human body are replaced with new cells every seven to 10 years. There is no denying that lessons from the past have advanced our species, but it may be the renewed cells talking, a voice inside us responding to a familiar risk with ‘it could be different this time’. It might be the optimist in us talking but we can sometimes convince ourselves that under different circumstances even an identical situation could have a different outcome.
Q.31) Which one of the following statements best reflects the most critical inference that can be made from the passage given above?
Correct
Solution (a)
Explanation
Option (a) is correct: The passage is about committing mistakes, sometimes repeatedly with optimism. This shows that humans keep on committing mistakes. The following lines – “As human beings, we go in cycles from one milestone to the next. Starting over is at the core of our existence. … There is no denying that lessons from the past have advanced our species, but it may be the renewed cells talking, a voice inside us responding to a familiar risk with ‘it could be different this time’”, show that even after making the mistakes we often start over again to make more mistakes. So, this is the best inference of the passage.
Option (b) is incorrect: The central theme of the passage is that irrespective of what we do, we commit mistakes due to various reasons. Consider the lines – “There is no denying that lessons from the past have advanced our species, but it may be the renewed cells talking, a voice inside us responding to a familiar risk with ‘it could be different this time’. It might be the optimist in us talking but we can sometimes convince ourselves that under different circumstances even an identical situation could have a different outcome.” So, this is not the correct inference of the passage.
Option (c) is incorrect: The context of reflection of ignorance due to the repetition of mistakes is not the central theme of the passage. Rather, the very idea of ignorance is not discussed in the passage. Hence, this is not the main inference of the passage.
Option (d) is incorrect: As per the passage, humans are optimistic despite making multiple mistakes. However, the given option states that an optimistic attitude helps humans prevent the repetition of mistakes, which is contrary to what is mentioned in the passage. Prevention of the repetition of mistakes is not a part of the passage. The author primarily talks about how committing mistakes, and even repeating the same mistake is intrinsic to human beings. So, this option is not the best inference.
Incorrect
Solution (a)
Explanation
Option (a) is correct: The passage is about committing mistakes, sometimes repeatedly with optimism. This shows that humans keep on committing mistakes. The following lines – “As human beings, we go in cycles from one milestone to the next. Starting over is at the core of our existence. … There is no denying that lessons from the past have advanced our species, but it may be the renewed cells talking, a voice inside us responding to a familiar risk with ‘it could be different this time’”, show that even after making the mistakes we often start over again to make more mistakes. So, this is the best inference of the passage.
Option (b) is incorrect: The central theme of the passage is that irrespective of what we do, we commit mistakes due to various reasons. Consider the lines – “There is no denying that lessons from the past have advanced our species, but it may be the renewed cells talking, a voice inside us responding to a familiar risk with ‘it could be different this time’. It might be the optimist in us talking but we can sometimes convince ourselves that under different circumstances even an identical situation could have a different outcome.” So, this is not the correct inference of the passage.
Option (c) is incorrect: The context of reflection of ignorance due to the repetition of mistakes is not the central theme of the passage. Rather, the very idea of ignorance is not discussed in the passage. Hence, this is not the main inference of the passage.
Option (d) is incorrect: As per the passage, humans are optimistic despite making multiple mistakes. However, the given option states that an optimistic attitude helps humans prevent the repetition of mistakes, which is contrary to what is mentioned in the passage. Prevention of the repetition of mistakes is not a part of the passage. The author primarily talks about how committing mistakes, and even repeating the same mistake is intrinsic to human beings. So, this option is not the best inference.
-
Question 32 of 35
32. Question
Passage – 2
Irregularity and lack of punctuality are problems which you encounter frequently in any work situation. Look around yourself and you may come across people who get up late in the morning and are unable to send their children to school on time. They may also not be able to provide their family members with proper meals and have disorganized homes due to their irregular behaviour and lack of punctuality. Such people are frequently absent from work. Others like to come in late and leave early as a matter of habit or right. Some workers are never found at their seats during working hours. You may have witnessed the inconvenience caused to the public wanting to pay bills when counter clerks are either late or not found at their seats in banks, post offices, and telephone and electricity offices. The irresponsible behaviour of such people spoils the discipline of the work environment, sets bad examples, causes inconvenience to the public and lowers the image of the organization.
Q.32) Which of the following statements can be inferred from the above passage?
- Professionalism and punctuality are crucial for a well-functioning work environment and public service.
- Personal actions have wider impacts beyond individual consequences.
Choose the correct answer using the codes given below.
Correct
Solution (c)
Explanation
Statement 1 is correct: The passage portrays the negative consequences of irregularity and lack of punctuality. These include disrupted teamwork, reduced efficiency, inconvenienced customers, and a damaged organizational image. The passage says, “Look around yourself and you may come across people who get up late in the morning and are unable to send their children to school on time. They may also not be able to provide their family members with proper meals and have disorganised homes due to their irregular behaviour and lack of punctuality. Such people are frequently absent from work”. This suggests that professionalism and adherence to expected work conduct are essential for maintaining a smooth-running environment at the workplace. Hence, statement 1 is correct.
Statement 2 is correct: The passage emphasizes how unprofessional behaviour ripples outward, affecting colleagues, customers, and the overall reputation of the organization. The passage says, “You may have witnessed the inconvenience caused to the public wanting to pay bills when counter clerks are either late or not found at their seats in banks, post offices, and telephone and electricity offices”. The passage further says, “The irresponsible behaviour of such people spoils the discipline of the work environment, sets bad examples, causes inconvenience to the public and lowers the image of the organization”. This implies that individuals hold a degree of responsibility for the collective success and image of their workplace, and their actions can have far-reaching consequences. Hence, statement 2 is correct.
Incorrect
Solution (c)
Explanation
Statement 1 is correct: The passage portrays the negative consequences of irregularity and lack of punctuality. These include disrupted teamwork, reduced efficiency, inconvenienced customers, and a damaged organizational image. The passage says, “Look around yourself and you may come across people who get up late in the morning and are unable to send their children to school on time. They may also not be able to provide their family members with proper meals and have disorganised homes due to their irregular behaviour and lack of punctuality. Such people are frequently absent from work”. This suggests that professionalism and adherence to expected work conduct are essential for maintaining a smooth-running environment at the workplace. Hence, statement 1 is correct.
Statement 2 is correct: The passage emphasizes how unprofessional behaviour ripples outward, affecting colleagues, customers, and the overall reputation of the organization. The passage says, “You may have witnessed the inconvenience caused to the public wanting to pay bills when counter clerks are either late or not found at their seats in banks, post offices, and telephone and electricity offices”. The passage further says, “The irresponsible behaviour of such people spoils the discipline of the work environment, sets bad examples, causes inconvenience to the public and lowers the image of the organization”. This implies that individuals hold a degree of responsibility for the collective success and image of their workplace, and their actions can have far-reaching consequences. Hence, statement 2 is correct.
-
Question 33 of 35
33. Question
A vessel contains mixture of petrol and diesel in the ratio 13: 5. 72 litres of the mixture is taken out of the vessel and replaced with 51 litres of diesel such that the ratio of the petrol to diesel in the vessel becomes 8: 7, respectively, then find the initial quantity of diesel in the vessel?
Correct
Solution (b)
Explanation
Let the initial quantities of petrol and diesel in the vessel be 13x litres and 5x litres respectively.
Quantity of petrol taken out = (13/18) × 72 = 52 litres
Quantity of diesel taken out = 72 – 52 = 20 litres
According to the question,
(13x – 52)/(5x – 20 + 51) = 8/7
91x – 364 = 40x + 248
51x = 612 x = 12
So, the initial quantity of diesel in the vessel = 12x = 12 × 5 = 60 litres Hence, option (b) is the correct answer.
Incorrect
Solution (b)
Explanation
Let the initial quantities of petrol and diesel in the vessel be 13x litres and 5x litres respectively.
Quantity of petrol taken out = (13/18) × 72 = 52 litres
Quantity of diesel taken out = 72 – 52 = 20 litres
According to the question,
(13x – 52)/(5x – 20 + 51) = 8/7
91x – 364 = 40x + 248
51x = 612 x = 12
So, the initial quantity of diesel in the vessel = 12x = 12 × 5 = 60 litres Hence, option (b) is the correct answer.
-
Question 34 of 35
34. Question
A thief got the following clue for the password of a locker:
- Three-digit code for the lock uses the digits from 0 to 9.
- The first digit cannot be either 0 or 1 and the second digit must be either 0 or 1.
- The second and third digits cannot both be 0.
How many different passwords are possible for the locker?
Correct
Solution (c)
Explanation
Total digits = 10
Let Code digits be X Y Z
Slot X can take 8 values, i.e. all except 0 & 1
Slot Y can take 2 values, either 0 or 1
If Slot Y takes digit “0”, then slot Z can only take 9 values (It can’t take digit zero)
If Slot Y takes digit “1”, then slot Z can take all values, i.e. all 10 digits.
Therefore, number of different passwords possible for the locker = (8 × 1 × 9) + (8 × 1 × 10) = 72 + 80 = 152.
Hence, option (c) is the correct answer.
Incorrect
Solution (c)
Explanation
Total digits = 10
Let Code digits be X Y Z
Slot X can take 8 values, i.e. all except 0 & 1
Slot Y can take 2 values, either 0 or 1
If Slot Y takes digit “0”, then slot Z can only take 9 values (It can’t take digit zero)
If Slot Y takes digit “1”, then slot Z can take all values, i.e. all 10 digits.
Therefore, number of different passwords possible for the locker = (8 × 1 × 9) + (8 × 1 × 10) = 72 + 80 = 152.
Hence, option (c) is the correct answer.
-
Question 35 of 35
35. Question
In a certain code ‘DIGEST’ is written as
‘GKICQW’. In the same code ‘PICNIC’ will be written as:
Correct
Solution (c)
Explanation
T → G (Opposite letter pair)
I + 2 → K
G + 2 → I
E – 2 → C
S – 2 → Q
D → W (Opposite letter pair) Similarly,
C → X (Opposite letter pair)
I + 2 → K
C + 2 → E
N – 2 → L
I – 2 → G
P → K (Opposite letter pair)
Hence, option (c) is the right answer.
Incorrect
Solution (c)
Explanation
T → G (Opposite letter pair)
I + 2 → K
G + 2 → I
E – 2 → C
S – 2 → Q
D → W (Opposite letter pair) Similarly,
C → X (Opposite letter pair)
I + 2 → K
C + 2 → E
N – 2 → L
I – 2 → G
P → K (Opposite letter pair)
Hence, option (c) is the right answer.
All the Best
IASbaba